Está en la página 1de 27

Pontificia Universidad Católica de Chile

Facultad de Fı́sica
FIS1533 - Electricidad y Magnetismo

Enunciado Ayudantı́a Transversal I2

18 de Noviembre 2023 - Cuerpo de Ayudantes FIS1533

Link Reunión: Link AT2

1 Módulo Alternativas y V/F


Ejercicio: Medios Conductores

Dos resistores cilı́ndricos están hechos del mismo material, con conductividad constante.
El resistor 2 es cuatro veces más largo y tiene la mitad de diámetro que el resistor 1.
¿Cuál es la relación entre sus resistencias?

a) R1 = 16R2
b) R1 = 2R2
c) 16R1 = R2
d) R1 = 4R2

Solución
Paso 0: ¿Cómo plantear el ejercicio?
Se pide determinar la relación entre las resistencias de resistores cilı́ndricos. Como se conoce
su resistividad, se utilizará la expresión de resistencia por definición:
Z
ρ
R= dl
A
Como el área se mantiene constante en relación al largo, no será necesario integrar.

1
Paso 1: Integral Resistencia - R1
En primer lugar, se arma la integral de resistencia para el resistor 1, considerando que las caras
del cuerpo son circunferencias de diámetro D:
Z
ρ
R1 = dl
A
Z L
ρ
= 2
dl
0 π(D/2)
Z L

= dl
πD2 0
4ρL
=
πD2

Paso 2: Integral Resistencia - R2


Luego, se arma la integral de resistencia para el resistor 2, considerando que las caras del
cuerpo son circunferencias de diámetro D/2:
Z
ρ
R2 = dl
A
Z 4L
ρ
= dl
0 π(D/4)2
16ρ 4L
Z
= dl
πD2 0
64ρL
=
πD2
Luego, se cumplirá la relación 16R1 = R2 .

2
Ejercicio: Ley de Ampère

Cuatro cables paralelos transportan cada uno una corriente I. Tres de los cables trans-
portan corriente hacia afuera de la página, mientras que uno transporta corriente hacia
al interior de la página. ¿Cuál es el sentido y dirección del campo magnético en el centro
del cuadrado?

Figura 1. Esquema sobre cables paralelos que transportan corriente.

a) 0 b) ↗ c) → d) ↘ e) ←

Solución
Paso Único: Principio de Superposición - Campos Magnéticos
Cada una de las corrientes generará un campo magnético de igual magnitud alrededor del
alambre (pues transportan una corriente de igual valor) y que experimentará eventualmente el
centro del cuadrado:

(a) Lı́neas de campo magnético en un


alambre (regla de mano derecha) (b) Campos magnéticos experimentados
en el centro del cuadrado
Figura 2. Diagramas sobre los campos magnéticos de los alambres.

Observando la figura, es posible determinar que el campo magnético total


experimentado en el centro del cuadrado será la suma vectorial de los cuatro campos
magnéticos individuales, o sea, apuntará hacia la esquina superior derecha.

3
Ejercicio: Fuerza de Lorentz

Una partı́cula positiva y negativa son introducidas en un campo magnético uniforme B ⃗


con cierta velocidad, tal como se indica en la figura. Ignore todo efecto gravitacional.

Figura 3. Esquema inicial sobre partı́culas cargadas en un campo magnético.

Lea las siguientes afirmaciones sobre la situación expuesta e indique si son verdaderas
(V) o falsas (F). Justifique solo las falsas.

a) F La partı́cula positiva empezará a describir un movimiento circunferencial



luego de estar un tiempo bajo el efecto del campo magnético B.
b) F La fuerza magnética sobre la partı́cula negativa debido al campo magnético
⃗ apuntará hacia la derecha (→).
B

c) V Ahora, suponga que el campo magnético B ⃗ apunta verticalmente hacia arriba


(↑). La fuerza magnética sobre la partı́cula negativa debido a este nuevo campo
magnético será cero.
d) F La magnitud de la velocidad de la partı́cula negativa cambiará luego de estar

un tiempo bajo el efecto del campo magnético uniforme B.
e) V Ahora, suponga que la velocidad de la partı́cula negativa se vuelve cero. La
⃗ será 0.
fuerza magnética sobre esta partı́cula debido al campo magnético B
f) F Ahora, suponga que no conoce ni el sentido ni dirección del campo magnético

B. Para que la trayectoria de la partı́cula positiva se deflecte verticalmente hacia
abajo (↓), es necesario que el campo magnético B ⃗ apunte hacia fuera de la página
(⊙).

4
Solución a)
Utilizando la expresión de fuerza de Lorentz para la partı́cula positiva:
F⃗L = q(E
⃗ + ⃗v × B)

⃗ = B ŷ y ⃗v = −v+ ŷ:
En ausencia de campos eléctricos y como q = +q, B

F⃗B = +q(−v+ ŷ × B ŷ)


=0
Como es un producto cruz entre dos componentes iguales, el resultado es 0. Como la fuerza
es nula, la partı́cula describirá una trayectoria rectilı́nea hacia la izquierda (←).

Solución b)
Utilizando la expresión de fuerza de Lorentz para la partı́cula negativa:
F⃗L = q(E
⃗ + ⃗v × B)

⃗ = B ŷ y ⃗v = v− ẑ:
En ausencia de campos eléctricos y como q = −q, B

F⃗B = −q(v− ẑ × B ŷ)


= qv− B x̂
Para realizar este producto cruz de manera sencilla se recuerda que estamos en la base “(x, y, z)”
en coordenadas cartesianas, por lo que ẑ × ŷ = −x̂. Al avanzar hacia atrás el signo es negativo.
Luego, la fuerza magnética apunta hacia fuera de la página (⊙).

Solución c)
Utilizando la expresión de fuerza de Lorentz para la partı́cula negativa:
F⃗L = q(E
⃗ + ⃗v × B)

⃗ = B ẑ y ⃗v = v− ẑ:
En ausencia de campos eléctricos y como q = −q, B

F⃗B = −q(v− ẑ × B ẑ)


=0
Como es un producto cruz entre dos componentes iguales, el resultado es 0.
Luego, la fuerza magnética es nula.

Solución d)
La partı́cula negativa está siendo afectada únicamente por una fuerza magnética. Estas no
realizan trabajo (i.e no alteran la rapidez o magnitud de la velocidad), pero sı́ modifican la
dirección y sentido del movimiento de la partı́cula.

5
Solución e)
Utilizando la expresión de fuerza de Lorentz para la partı́cula negativa:
F⃗L = q(E
⃗ + ⃗v × B)

⃗ = B ẑ y ⃗v = 0:
En ausencia de campos eléctricos y como q = −q, B

F⃗B = −q(0 × B ẑ)


=0
Como la velocidad es nula, la fuerza magnética también lo será.

Solución f )
Utilizando la expresión de fuerza de Lorentz para la partı́cula positiva:
F⃗L = q(E
⃗ + ⃗v × B)

⃗ = B ˆ? y ⃗v = −v+ ŷ:
En ausencia de campos eléctricos y como q = +q, B

F⃗B = +q(−v+ ŷ × B ˆ?)


Se requiere que la partı́cula deflecte verticalmente hacia abajo (i.e F⃗B = −FB ẑ),
por lo que se debe cumplir que:

−ẑ = −ŷ × ˆ?
ˆ? = −x̂

Para realizar este producto cruz de manera sencilla se recuerda que estamos en la base “(x, y, z)”
en coordenadas cartesianas, por lo que ŷ × x̂ = −ẑ. Al avanzar hacia atrás el signo es negativo.
Luego, el campo magnético debe apuntar hacia al interior de la página (⊗).
Nota: El ejercicio también se pudo haber resuelto con la ayuda de la regla de la mano derecha, notando
que para partı́culas negativas se realiza un ajuste de inversión para las variables. Además, se consideró
el siguiente sistema de referencia para los cálculos realizados:

Figura 4. Sistema de coordenadas cartesianas.

6
Ejercicio: Ley de Faraday-Lenz

Considere las dos bobinas de la figura.

Figura 5. Esquema sobre las bobinas.

Suponga que se hacen los siguientes experimentos independientes con ellas:

I. El interruptor S está cerrado y se abre


II. El interruptor S está abierto y se cierra
III. Con el interruptor cerrado A se aleja de B
IV. Con el interruptor cerrado A se acerca a B

¿Para cuáles de los cuatro experimentos anteriores la corriente I que pasa por la re-
sistencia R circulará en el sentido mostrado en la figura?

a) En los experimentos I y IV
b) En los experimentos II y III
c) Solamente en el experimento II
d) Solamente en el experimento I

7
2 Módulo Desarrollo
Ejercicio: Condensadores y Medios Dieléctricos

Se tiene una esfera de material dieléctrico de permitividad ε1 = 5ε0 . La esfera está


rodeada de un conductor ideal cargado con carga Q que a su vez está rodeado de un
material dieléctrico de permitividad ε2 = 10ε0 . Finalmente, en la última capa hay un
conductor con carga −Q. Los radios de cada una de las capas esféricas son R1 = R,
R2 = 2R, R3 = 3R y R4 = 4R, respectivamente.

Figura 6. Esquema sobre la configuración esférica.

a) Determine los campos eléctricos al interior del dieléctrico 1 y 2.


b) ¿Cuál es la capacitancia de la configuración?
c) Calcule la carga de polarización en la superficie del dieléctrico 1.
d) Calcule la carga de polarización en la superficie externa del dieléctrico 2.

Solución a)
Paso 0: ¿Cómo plantear el ejercicio?
Se pide determinar el campo eléctrico en los dieléctricos 1 y 2. Como se está hablando de una
esfera y de materiales dieléctricos, es posible aplicar Ley de Gauss Generalizada utilizando una
esfera concéntrica de radio “r” arbitrario:
I
D⃗ · d⃗s = Qf

Ahora, el vector desplazamiento para un dieléctrico está dado por:


⃗ = εE
D ⃗ = ε0 κ E

Además, y como se presentan “cortes visuales” relacionados con un cambio en la densidad de carga,
se debe trabajar por intervalos: r < R1 , R1 < r < R2 , R2 < r < R3 , R3 < r < R4 y r > R4 .

8
⃗ 0<r<R
Paso 1: Ley de Gauss Generalizada - D 1

En primer lugar, se calculará el desplazamiento eléctrico en la región 0 < r < R1 :


I
⃗ 0<r<R · dS
D ⃗ = Qf
1

Se conoce que el desplazamiento solo posee componente radial.


Además, no hay carga encerrada libre en esta región:
⃗ 0<r<R = Qf = 0
∴D (1)
1

⃗ 0<r<R
Paso 2: Campo Eléctrico Dieléctricos - E 1

Luego, el vector desplazamiento está dado por:


⃗ = εE
D ⃗ = ε0 κ E

Es importante notar que se puede ocupar cualquiera de las dos fórmulas anteriores.
Por conveniencia se utilizará la primera, pues por enunciado dan el dato de “ε”.
Despejando el campo eléctrico, se obtiene que:
⃗ 0<r<R = εE
D ⃗ 0<r<R
1 1


∴ E0<r<R = 0 (2)
1

⃗ R <r<R
Paso 3: Ley de Gauss Generalizada - D 2 3

En primer lugar, se calculará el desplazamiento eléctrico en la región R2 < r < R3 :


I
⃗ R <r<R · dS
D ⃗ = Qf
2 3

Se conoce que el desplazamiento solo posee componente radial.


Además, la carga encerrada libre será la carga total Q del conductor 1:
I
DR2 <r<R3 · dS = Q

Ahora, la integral cerrada es en verdad la superficie de la esfera,


pues el flujo es ortogonal a esta en todo momento:

DR2 <r<R3 · 4πr2 = Q


Q
DR2 <r<R3 =
4πr2
Aplicando la componente radial:

⃗ R <r<R = Q
∴D 2 3 r̂ (3)
4πr2

9
⃗ R <r<R
Paso 4: Campo Eléctrico Dieléctricos - E 2 3

Luego, el vector desplazamiento está dado por:


⃗ = εE
D ⃗ = ε0 κ E

Es importante notar que se puede ocupar cualquiera de las dos fórmulas anteriores.
Por conveniencia se utilizará la primera, pues por enunciado dan el dato de “ε”.
Despejando el campo eléctrico, se obtiene que:
⃗ R <r<R = εE
D ⃗ R <r<R
2 3 2 3

∴E ⃗ R <r<R = Q r̂ (4)
2 3
4πr2 ε2
Se usó “ε2 ” debido a que es la permitividad del dieléctrico que se encuentra en la región 0 < r < b.
Reemplazando valores se obtiene que (i.e ε2 = 10ε0 ):

⃗ R <r<R = Q
∴E 2 3 r̂ (5)
40πr2 ε0

Solución b)
Paso 0: ¿Cómo plantear el ejercicio?
Se pide determinar la capacitancia del sistema. Para ello, es necesario solo conocer el campo
y potencial eléctrico que se encuentra en medio de los lugares que contienen cargas opuestas
pero de igual magnitud (i.e R2 < r < R3 ). Como se está hablando de una esfera, es posible
aplicar Ley de Gauss utilizando una esfera concéntrica de radio “r” arbitrario:
I
⃗ · d⃗s = Qenc
E
S ε0
Luego, se puede utilizar la forma integral de la relación campo - potencial:
Z ⃗rb
V (⃗rb ) − V (⃗ra ) = − ⃗ · d⃗l
E

ra

Además, y como se presentan “cortes visuales” relacionados con un cambio en la densidad de carga,
se debe trabajar por intervalos: r < R1 , R1 < r < R2 , R2 < r < R3 , R3 < r < R4 y r > R4 .
Finalmente, se ocupa la fórmula que relaciona carga almacenada con capacitancia:
Q
Q = C∆V → C =
∆V

Paso 1: Potencial Eléctrico - ∆V


Ahora, se calculará la diferencia de potencial eléctrico entre r = R2 y r = R3 :
Z R2
∆V = V (R2 ) − V (R3 ) = − E⃗ r>R · d⃗l
2
R3

Por propiedades de integrales se pueden invertir los lı́mites


y la única componente en juego es la radial, por lo que d⃗l = d⃗r:

10
Z R3
Q
∆V = V (R2 ) − V (R3 ) = dr
R2 40πr2 ε0
Q 1 R3
= (− )
40πε0 r R2
Q 1 1
= ( − )
40πε0 R2 R3
Reemplazando valores se obtiene que:
Q 1 1 Q
∴ ∆V = ( − )= (6)
40πε0 2R 3R 240πε0 R

Paso 2: Capacitancia - C
Finalmente, se ocupa la fórmula que relaciona carga almacenada con capacitancia:
Q
C=
∆V
Q
= Q
240πε0 R
= 240πε0 R

Solución c) y d)
Paso 0: ¿Cómo plantear el ejercicio?
Se pide determinar la carga de polarización en la superficie del dieléctrico interno ε1 y en la
superficie externa del dieléctrico ε2 (i.e r = R1 y r = R3 ). Luego, se debe utilizar que:

σP (⃗x) = P⃗ (⃗x) · n̂(⃗x)


r=a

De aquı́ se nota que falta el vector polarización, el cual está dado por:

P⃗ (⃗x) = (ε − ε0 )E(⃗
⃗ x)

Luego, la carga en la superficie estará determinada por una integral en esféricas:


ZZ
Q= σ dS
S

Paso 1: Vector Polarización - P⃗0<r<R1


Ahora, se calculará el vector polarización en la región 0 < r < R1 :
P⃗0<r<R1 = (ε1 − ε0 )E
⃗ 0<r<R
1
=0
Se usó “ε1 ” debido a que es la permitividad del dieléctrico que se encuentra en la región 0 < r < R1 .

11
Paso 2: Densidad Carga Polarización Superficial - σPR1
Posteriormente, se utiliza la expresión de densidad de carga de polarización superficial evaluada
en el lı́mite r = R1 :

σPR1 = P⃗0<r<R1 · n̂
r=R1

En este caso n̂ apunta hacia afuera de forma radial (i.e n̂ = r̂), puesto que se dirige
hacia el exterior del dieléctrico. Reemplazando el vector polarización se obtiene que:
∴ σPR1 = 0

Paso 3: Carga Polarización Superficial - QPR1


Ası́, la carga en la superficie R1 estará determinada por una integral en esféricas:
ZZ
Q= σ dS
S

Como la densidad de carga del cascarón es uniforme,


se puede tratar como una constante y sale de la integral.
Su radio es R1 y su densidad de carga es 0:

∴ QPR1 = 0 (7)

Paso 4: Vector Polarización - P⃗R2 <r<R3


Ahora, se calculará el vector polarización en la región R2 < r < R3 :
P⃗R2 <r<R3 = (ε2 − ε0 )E
⃗ R <r<R
2 3

Q(ε2 − ε0 )
= r̂
40πr2 ε0
Se usó “ε2 ” debido a que es la permitividad del dieléctrico que se encuentra en la región R2 < r < R3 .

Paso 5: Densidad Carga Polarización Superficial - σPR3


Posteriormente, se utiliza la expresión de densidad de carga de polarización superficial evaluada
en el lı́mite r = R3 :

σPR3 = P⃗R2 <r<R3 · n̂


r=R3

En este caso n̂ apunta hacia afuera de forma radial (i.e n̂ = r̂), puesto que se dirige
hacia el exterior del dieléctrico. Reemplazando el vector polarización se obtiene que:

Q(ε2 − ε0 )
σPR3 = r̂ · r̂
40πr2 ε0 r=R3
Q(ε2 − ε0 )
=
40πR32 ε0

12
Paso 6: Carga Polarización Superficial - QPR3
Ası́, la carga en la superficie R3 estará determinada por una integral en esféricas:
ZZ
Q= σ dS
S

Como la densidad de carga del cascarón es uniforme,


se puede tratar como una constante y sale de la integral.
Su radio es R3 y su densidad de carga es σPR3 :

Z 2π Z π
QPR3 = σPR3 R32 sin θ dθdϕ
0 0
Q(ε2 − ε0 )
= · 2π · 2 · R32
40πR32 ε0
Q(ε2 − ε0 )
=
10ε0
Reemplazando valores se obtiene que (i.e ε2 = 10ε0 ):
9Q
∴ QPR3 = (8)
10
Nota: El vector polarización solo tiene sentido cuando se habla de algún material dieléctrico, por lo
que la región propicia para calcularlo es en donde este se ubica. Además, no era estrictamente necesario
realizar la integral de superficie para determinar la densidad de carga. Bastaba con saber que se referı́a
al área de una esfera de radio Ri , con i = 1, 3.

13
Ejercicio: Introducción a Circuitos Eléctricos - Circuitos RC
El circuito de la figura ha estado operando por mucho tiempo, hasta que en t = 0 un interruptor
cambia su topologı́a.

Figura 7. Esquema del circuito original RC.

14
a) Reduzca el circuito de manera que quede de la siguiente forma:

Figura 8. Esquema del circuito reducido RC.

¿Cómo se puede expresar Ra y Rb en términos de parámetros conocidos?


b) ¿Cómo se puede expresar el voltaje del resistor Rb según la notación de la figura 8 y el
voltaje del capacitor VC ?
c) Calcule la carga almacenada por el capacitor en t = 0 según la notación de la figura 8.
d) Determine la diferencia de potencial para t > 0 según la notación de la figura 8.
e) Determine la intensidad de corriente que circula por el capacitor para t > 0 según la
notación de la figura 8.
f) ¿Cuál es la constante de tiempo capacitiva luego de que cambia la topologı́a del circuito?
g) Encuentre la nueva diferencia de potencial en el capacitor según la notación de la figura
8, luego de que ha transcurrido mucho tiempo con el cambio de topologı́a.
h) Suponga que entre las placas del condensador se inserta un material dieléctrico de per-
mitividad eléctrica ε = 2ε0 . ¿Cómo esto afecta a los tiempos de carga y descarga del
capacitor? Considere que en un principio solo se presentaba vacı́o entre las placas.

15
Solución a)
Paso Único: Resistencia Equivalente
Se pide reducir el circuito hasta obtener la forma deseada. Es posible observar que se produce un
cortocircuito en la parte superior del circuito, puesto que una rama sin ningún componente circuital
está conectada en paralelo con otras ramas (i.e R1 ). La corriente preferirá circular por aquel camino
que menos se le oponga, por lo que el circuito queda como:

Figura 9. Esquema sobre la reducción de resistencias del circuito.

Del esquema, se nota que R2 está conectada en paralelo con R3 :

Req,1 = R2 R3
Asimismo, se nota que R2 R3 está conectada en serie con R4 y R5 :
∴ Ra = (R2 R3 ) + R4 + R5 (9)
Finalmente, se nota que R6 está conectada en paralelo con R7 :
∴ Rb = R6 R7 (10)

16
Solución b)
Paso 0: ¿Cómo plantear el ejercicio?
Se pide determinar la carga almacenada por el capacitor en t = 0. Si el circuito ha estado operando
en este estado durante mucho tiempo, la fuente de voltaje V1 entregará una constante alimentación al
capacitor y, eventualmente, alcanzará su carga máxima. En este punto su carga ya no puede variar,
por lo que se comportará como circuito abierto:
dQ
IC = =0
dt
Finalmente, se ocupa la fórmula que relaciona carga almacenada con capacitancia:
Q = CV

Paso 1: Condiciones Iniciales - VC (0)


Es posible observar que se produce un circuito abierto tanto por el interruptor como por el capacitor
en la parte central del circuito. La corriente eléctrica no puede circular por tales ramas, por lo que el
circuito queda como:

Figura 10. Esquema sobre las condiciones iniciales del circuito.

Luego, el voltaje entre los terminales del capacitor estará dado por:

VC (0) = −V1
Finalmente, se ocupa la fórmula que relaciona carga almacenada con capacitancia:
Q(0) = CVC (0)
= −CV1

17
Solución c), d) y e)
Paso 0: ¿Cómo plantear el ejercicio?
Se pide determinar la diferencia de potencial y la intensidad de corriente que circula por el capacitor
para t > 0, o sea, al cambiar la topologı́a. Luego, se deberá resolver el circuito mediante algún método,
de forma de armar la EDO respectiva:
dQ dVC
IC = =C
dt dt
Además, se deben aplicar las condiciones iniciales en relación al capacitor.

Paso 1: Preparación KVL


Para t > 0, el circuito queda como:

Figura 11. Esquema KVL del circuito (cambio topologı́a).

Como se presenta una única malla, se preferirá realizar este ejercicio mediante ese método.

Paso 2: Resolución KVL - IA e IB


Para armar las ecuaciones de KVL debemos observar una malla y recorrerla en el sentido otorgado. Al
pasar por un elemento circuital, se sumará su voltaje si se entra de + a − (en caso contrario se resta):

Malla Ecuación KVL

A −VC + VRb + V2 = 0

Luego, los voltajes provenientes de resistores son reemplazados mediante Ley de Ohm:

Malla Ecuación KVL

A −VC + V2 + IA Rb = 0

Para obtener la EDO se aplicará la expresión IC = −IA = C dV


dt :
C

dVC
∴ −VC + V2 − CRb =0 (11)
dt

18
Paso 3: Resolución EDO - VC (t)
La EDO se puede tratar mediante el método de separación de variables, por lo que se dejará a un lado
de la ecuación solo variables dependientes de “VC ” y al otro solo variables dependientes del tiempo y/o
constantes:
dVC
−VC + V2 − CRb =0
dt
dt dVC
=
CRb V2 − VC
Integrando con condiciones iniciales generales:
Z t Z VC
dt dVC
=
t0 CRb VC (0) V2 − VC

El interruptor cambia de posición en t = 0 (i.e t ∈ [0, ∞[).


Asimismo, el condensador estará sometido a cierta diferencia de potencial inicial VC (0) = −V1 :
Z t Z VC
dt dVC
=
0 CR b −V1 2 − VC
V
Z V2 −VC
t du
− =
CRb V2 +V1 u
t V2 − VC
− = ln ( )
CRb V2 + V1
Notar que se realizó un cambio de variable u = V2 − VC → du = −dVC .
Por último, despejamos VC aplicando exponencial:
t
− CR V2 − VC
e b =
V2 + V1
t
− CR
∴ VC (t) = V2 − (V2 + V1 )e b (12)

Paso 5: Intensidad Corriente Capacitor - IC (t)


Utilizando la expresión que relaciona la intensidad de corriente con el voltaje en el capacitor:
dVC (t)
IC (t) = C
dt
d − t
= C (V2 − (V2 + V1 )e CRb )
dt
− t 1
= −C(V2 + V1 )e CRb · −
CRb
(V2 + V1 ) − CRt
= e b
Rb

Paso 4: Voltaje Capacitor Infinito - VC (∞)


Además, y si transcurre mucho tiempo luego de que el interruptor se cierra (i.e t → ∞), la diferencia
de voltaje en el capacitor será:
VC (∞) = lim VC (t)
t→∞
t
− CR
= lim V2 − (V2 + V1 )e b
t→∞
= V2

19
Paso 5: Constante de Tiempo Capacitiva - τ
Observando y comparando lo obtenido, la constante de tiempo capacitiva vendrá dada por:
t t
− =−
τ CRb
τ = CRb

Solución f )
Paso Único: Modificación Permitividad - τ
Se pide determinar cualitativamente los cambios que experimentan los tiempos de carga y descarga del
capacitor cuando se modifica la permitividad del material que se encuentra entre sus placas. En primer
lugar, la capacitancia de un condensador de placas paralelas se puede escribir como:
εA
C=
d
Considerando que el condensador no sufre ningún otro cambio
a excepción de la permitividad del material entre las placas:
ε0 A
C1 =
d
εA 2ε0 A
C2 = =
d d
Luego, la relación entre la capacitancia inicial y final vendrá dada por:
C2 = 2C1
Se nota que un cambio en la capacitancia afectará directamente a la constante de tiempo capacitiva:
τ1 = C1 Rb
τ2 = C2 Rb = 2C1 Rb
Luego, la relación entre la constante de tiempo capacitiva inicial y final vendrá dada por:
τ2 = 2τ1
Finalmente, concluimos que la constante de tiempo capacitiva aumenta al doble y, con ello,
los tiempos de carga y/o descarga se incrementan con respecto a la situación inicial.

20
Ejercicio: Ley de Biot-Savart
Un alambre tiene la forma de una espiral de Arquı́medes dada por:
b
r(θ) = a + θ, 0≤θ≤π
π
Donde θ es el ángulo con respecto al eje x,
más un trozo recto que une los puntos a y −(a + b).
Considere que la componente +ẑ apunta hacia fuera de la página.

Figura 12. Esquema sobre el alambre en estudio.

a) Si el alambre lleva una corriente uniforme I, determine el campo magnético en el punto


O generado por el segmento polar.
b) Si el alambre lleva una corriente uniforme I, determine el campo magnético en el punto
O generado por el segmento recto.
c) Si el alambre lleva una corriente uniforme I, determine el campo magnético total en el
punto O.

Solución a), b) y c)
Paso 0: ¿Cómo plantear el ejercicio?
Se pide determinar el campo magnético en el punto O. Como la figura no posee ninguna simetrı́a apta
para la Ley de Ampére, solo queda aplicar campo por definición:
Id⃗l × (⃗r − ⃗r ′ )
Z
⃗ r ) = µ0
B(⃗
4π L |⃗r − ⃗r ′ |3
Se nota que se usa una integral simple debido a la naturaleza del problema,
puesto que el alambre vive en una dimensión. Finalmente, el campo magnético en el
punto O estará dado por la suma de los campos generados por el segmento recto y polar.

⃗ P (0)
Paso 1: Campo Magnético Segmento Polar - B
Desde ahora en adelante, nuestro origen de referencia será el punto O y se utilizarán coordenadas
cilı́ndricas por la geometrı́a del ejercicio. Se recuerda que el vector ⃗r parte en el origen y se extiende
hasta el punto en donde se quiere calcular el campo:

21
⃗r = 0
Como el origen de referencia coincide con el punto a calcular el vector es nulo.
Luego, el vector ⃗r ′ parte en el origen y recorre todo el cuerpo con corriente:
b
⃗r ′ = rr̂, θ, θ ∈ [0, π]
r(θ) = a +
π
Donde “r” es el radio del espiral y “r̂” la componente radial en coordenadas cilı́ndricas.
Se siguen determinando componentes de la integral:
⃗r − ⃗r ′ = −rr̂
Calculando el módulo del vector anterior:
|⃗r − ⃗r ′ | = r
Considerando que la corriente es lineal y que circula en sentido θ̂:
d⃗l = r dθθ̂
Para llegar a lo anterior se utilizó el diferencial de lı́nea
para cilı́ndricas, observando la componente variable.
Utilizando los resultados anteriores:
Id⃗l × (⃗r − ⃗r ′ ) = Irdθθ̂ × −rr̂
= Ir2 dθ ẑ
Para realizar este producto cruz de manera sencilla se recuerda que estamos en la base “(r, θ, z)”
en coordenadas cilı́ndricas, por lo que θ̂ × r̂ = −ẑ. Al avanzar hacia atrás el signo es negativo.
Finalmente, se arma la integral del campo magnético para el segmento polar:
Id⃗l × (⃗r − ⃗r ′ )
Z
B⃗ P (0) = µ0
4π L |⃗r − ⃗r ′ |3
Z π 2
µ0 Ir dθ ẑ
=
4π 0 r3
Z π
µ0 Idθ ẑ
=
4π 0 r
µ0 I π dθ ẑ
Z
=
4π 0 a + πb θ
µ0 I a+b π du ẑ
Z
=
4π a b u
a+b
µ0 I
= (ln u) ẑ
4b a
µ0 I
= (ln (a + b) − ln a) ẑ
4b
µ0 I a+b
= ln ( ) ẑ
4b a
µ0 I b
= ln (1 + ) ẑ
4b a
Notar que se realizó un cambio de variable u = a + πb θ → du = b
π dθ.

22
⃗ R (0)
Paso 2: Campo Magnético Segmento Recto - B
Desde ahora en adelante, nuestro origen de referencia será el punto “O” y se utilizarán coordenadas
cartesianas por la geometrı́a del ejercicio. Se recuerda que el vector ⃗r parte en el origen y se extiende
hasta el punto en donde se quiere calcular el campo:
⃗r = 0
Como el origen de referencia coincide con el punto a calcular el vector es nulo.
Luego, el vector ⃗r ′ parte en el origen y recorre todo el cuerpo con corriente:
⃗r ′ = xx̂, x ∈ [−(a + b), a]
Donde “r” es el largo del segmento y “r̂” la componente radial en coordenadas cilı́ndricas.
Se siguen determinando componentes de la integral:
⃗r − ⃗r ′ = 0 − xx̂
Calculando el módulo del vector anterior:
|⃗r − ⃗r ′ | = x
Considerando que la corriente es lineal y que circula en sentido x̂:
d⃗l = dxx̂
Para llegar a lo anterior se utilizó el diferencial de lı́nea
para cartesianas, observando la componente variable.
Utilizando los resultados anteriores:
Id⃗l × (⃗r − ⃗r ′ ) = Idxx̂ × −xx̂
=0
Como es un producto cruz entre dos componentes iguales, el resultado es 0.
Finalmente, se arma la integral del campo magnético para el segmento recto:
Id⃗l × (⃗x − ⃗x ′ )
Z
⃗ µ0
BR (0) =
4π L |⃗x − ⃗x ′ |3
=0


Paso 3: Principio de Superposición - B(0)
Finalmente, el campo magnético en el punto O será la suma de los campos generados por los 2 segmentos:

B(0) =B⃗ P (0) + B⃗ R (0)
µ0 I b
= ln (1 + ) ẑ
4b a

23
Ejercicio: Ley de Ampére y Superposición
Considere tres distribuciones como se muestra en la figura. Un bloque de ancho w lleva una
densidad volumétrica uniforme de corriente J⃗1 = J1 ẑ. Un cable infinito de radio R lleva una
densidad de corriente no uniforme J⃗2 = −αrẑ, donde r es la distancia desde el eje del cilindro
y α una constante. El bloque se encuentra a una distancia d del cilindro. Se coloca un alambre
con corriente I a una distancia s del centro del cable (s < d).

Figura 13. Esquema sobre las distribuciones en estudio.

a) Calcule el campo magnético generado por el bloque de ancho w fuera de él.


b) Calcule el campo magnético generado por el cilindro de radio R fuera de él.
c) Determine la fuerza por unidad de largo que siente el cable con corriente I.

Solución a)
Paso 0: ¿Cómo plantear el ejercicio?
Se pide determinar el campo magnético fuera del bloque de ancho w. Como se está hablando de un
bloque de extensión infinita, es posible aplicar Ley de Ampére con un circuito rectangular centrado en
la mitad del bloque en sentido antihorario de largo “l” arbitrario:
I
⃗ · d⃗l = µ0 Ienc
B
L

No se entrega explı́citamente la corriente contenida en el bloque,


por lo que se tendrá que utilizar la definición de corriente:
ZZ
I= ⃗ x) · dS(⃗
J(⃗ ⃗ x)
S

24
Paso 1: Ley de Ampére - B
En primer lugar, se calculará el campo magnético fuera del bloque de ancho w:
I
⃗ · d⃗l = µ0 Ienc
B
L

Se conoce que el campo solo posee componente angular y que J⃗ = J ẑ, por lo que
se utilizará el diferencial de superficie que contiene a “ẑ” en coordenadas cilı́ndricas.
Además, la corriente encerrada estará determinada por una integral en cilı́ndricas:
I ZZ
Br>R · dl = µ0 J1 ẑ · dxdy ẑ
S
Como la densidad de corriente del plano es uniforme,
se puede tratar como una constante y sale de la integral.
Ahora, la integral cerrada son en verdad los lados de largo “l” horizontales
del circuito ampèriano, pues el flujo es paralelo a estos en todo momento:
ZZ
Br>R · 2l = µ0 J1 dxdy
S
La integral de superficie se refiere al área encerrada por el circuito ampèriano y que
contiene corriente. En este caso, serı́a el área de un rectángulo de largo “l” y de ancho “w”:
Br>R · 2l = µ0 J1 wl
µ0 J1 w
Br>R =
2
Se debe notar que los otros lados no contribuyen,
debido a que se anulan entre ellos y en cada lugar del bloque.


Paso 2: Ley de Ampére - B
Se conoce que el campo solo posee componente en el eje x, pero su sentido variará dependiendo del
cuadrante en donde se encuentre. Esto se puede observar gracias a la regla de la mano derecha,
posicionando el pulgar como corriente y los demás dedos como el campo magnético:

⃗ y<−w/2 = µ0 J1 w x̂
∴B (13)
2
⃗ y>w/2 = − 0 1 w x̂
∴B
µ J
(14)
2

Solución b)
Paso 0: ¿Cómo plantear el ejercicio?
Se pide determinar el campo magnético fuera del cilindro de radio R. Como se está hablando de un
cilindro de extensión infinita, es posible aplicar Ley de Ampére con una curva concéntrica circunferencial
de radio “r” arbitrario:
I
B⃗ · d⃗l = µ0 Ienc
L
No se entrega explı́citamente la corriente contenida en el cilindro,
por lo que se tendrá que utilizar la definición de corriente:
ZZ
I= ⃗ x) · dS(⃗
J(⃗ ⃗ x)
S
Además, y como se presentan “cortes visuales” relacionados con un cambio
en la densidad de corriente, se debe trabajar por intervalos: r < R y r > R.

25
⃗ r>R
Paso 1: Ley de Ampére - B
Ahora, se calculará el campo magnético en la región r > R:
I
⃗ r>R · d⃗l = µ0 Ienc
B

Se conoce que el campo solo posee componente angular y que J⃗ = J ẑ, por lo que
se utilizará el diferencial de superficie que contiene a “ẑ” en coordenadas cilı́ndricas.
Además, la corriente encerrada estará determinada por una integral en cilı́ndricas:
I Z 2π Z R
Br>R · dl = µ0 J2 (r) ẑ · r drdθ ẑ
0 0
Z 2π Z R
= µ0 −αr · r drdθ
0 0

Ahora, la integral cerrada es en verdad el perı́metro de la circunferencia,


pues el campo es paralelo a este en todo momento:
Z R
Br>R · 2πr = µ0 · 2π −αr2 dr
0
R3
Br>R · 2πr = −µ0 · 2π · α
3
µ0 αR3
Br>R = −
3r
Aplicando la componente angular:
3
⃗ r>R = − µ0 αR θ̂
∴B (15)
3r

Solución c)
Paso 0: ¿Cómo plantear el ejercicio?
Se pide determinar la fuerza magnética por unidad de largo que experimenta el cable con corriente I:
Z
FB = Id⃗l × B
⃗ ⃗

Este cuerpo vive en una dimensión, de modo que se debe usar una integral simple.

Paso 1: Evaluación Campos Magnéticos


En primer lugar, debemos notar cómo afectan los campos magnéticos calculados al alambre. En el caso
del plano, y como el alambre se posiciona por debajo de este, solo nos importará:

⃗ y<−w/2 = µ0 J1 w x̂
B
2
Este campo es constante e independiente de la posición,
por lo que no es necesario evaluarlo.
Considerando que el campo magnético generado por el cilindro apunta hacia la izquierda para cada
punto del alambre (por regla de la mano derecha) y que se ubica a una distancia “s” del eje del mismo,
se debe evaluar su campo en r = s:
3
⃗ r>R (s) = µ0 αR x̂
∴B (16)
3s

26
Nota: Se recuerda que el campo de cuerpos cilı́ndricos se origina desde su propio eje. Es importante
notar que no es necesario calcular el campo magnético en las demás regiones para el cilindro, debido a
que el punto a evaluar se encuentra en la región r > R.

Paso 2: Componentes Integral Fuerza - Id⃗l × B



Considerando que la corriente del alambre es lineal y que circula en sentido ẑ:
d⃗l = dz ẑ
Para llegar a lo anterior se utilizó el diferencial de lı́nea
para cartesianas, observando la componente variable.
Por principio de superposición y utilizando los resultados anteriores:
3
⃗ = Idz ẑ × ( µ0 J1 w x̂ + µ0 αR x̂)
Id⃗l × B
2 3s
µ0 J1 w µ0 αR3
= I( + )dz ŷ
2 3s
Para realizar este producto cruz de manera sencilla se recuerda que estamos en la base “(x, y, z)”
en coordenadas cartesianas, por lo que ẑ × x̂ = ŷ. Al avanzar hacia adelante el signo es positivo.

Paso 3: Integral Fuerza Magnética - F⃗B


Finalmente, se arma la integral de la fuerza magnética, considerando que el alambre posee una longitud
“L” arbitraria:
Z
F⃗B = Id⃗l × B ⃗
L
µ0 J1 w µ0 αR3
Z
F⃗B = I( + )dz ŷ
0 2 3s
µ0 J1 w µ0 αR3
F⃗B = L · I( + ) ŷ
2 3s
F⃗B µ0 J1 w µ0 αR3
= I( + ) ŷ
L 2 3s

27

También podría gustarte